Вы находитесь на странице: 1из 15

A 60-year-old woman presents to the emergency room with 12 hours of chest pain.

Her history is notable for hypertension, diabetes, and a hysterectomy a year ago. In the emergency department, her blood pressure is 145/90 mm Hg and heart rate is 92 beats/min. Her lungs are clear. An electrocardiogram shows 2-mm ST-segment elevations anteriorly. Which of the following is true? A. She should not receive streptokinase because of her hysterectomy a year ago B. Tissue plasminogen activator (tPA) is contraindicated because of her risk factors for stroke C. She should be treated with aspirin, heparin, and -blockers, but she is not a candidate for thrombolytics D. The catheterization team should be called, but if they are not readily available, she should be given tPA E. The catheterization team should be called, but if they are not readily available, she should be given a glycoprotein IIb/IIIa (GP IIb/IIIa) inhibitor
Correct Answer: The catheterization team should be called, but if they are not readily available, she should be given tPA This woman is having an acute anterior ST-segment elevation myocardial infarction (STEMI) and needs immediate reperfusion therapy. Given that she is presenting late, at 12 hours, the first choice, if possible, is primary angioplasty. If this procedure cannot be performed quickly, she has no absolute contraindications to thrombolytic therapy and should receive it as soon as possible (thus Answer C is incorrect). Answer A is incorrect because surgery (hysterectomy) a year ago is not a contraindication to thrombolytics (either streptokinase or tPA); likewise, the presence of risk factors for stroke (Answer B) is also not a contraindication. Answer E is incorrect because GP IIb/IIIa inhibitors are not indicated for STEMI, only for nonST-segment elevation myocardial infarction and unstable angina.

A 62-year-old man presents to the emergency room with substernal, crushing chest pain for 30 minutes. He is diaphoretic and short of breath. His only past medical history is that he suffered a GI bleed from peptic ulcer disease 6 months ago and has had no recurrence. Physical examination is notable only for a blood pressure (BP) of 160/95 mm Hg. An electrocardiogram shows STsegment elevation of 2 mm in leads V1 through V4. The nearest catheterization lab is at a hospital approximately 2 hours away. Which is the next appropriate action? A. He should be transported immediately for catheterization because reperfusion success rate is greater with percutaneous coronary intervention (PCI) than with thrombolysis B.

He should not be given thrombolytics because he has had a recent GI bleed C. He should be given thrombolytics because the catheterization lab is too far away and he has no absolute contraindications D. He should not be given thrombolytics because his systolic blood pressure is too high and places him at risk for hemorrhage E. He should be referred for emergent catheterization because he is in cardiogenic shock

Correct Answer: He should be given thrombolytics because the catheterization lab is too far away and he has no absolute contraindications This question addresses the indications and contraindications of both emergent PCI and thrombolytics in a patient with an acute ST-segment elevation myocardial infarction. The correct answer is to give thrombolytics immediately because of the distance to the nearest catheterization lab. Answer A is incorrect; while it is true that PCI has a higher success rate than thrombolytics in opening an artery, thrombolytics should be given if it will take more than 90 minutes to perform PCI and if there are no contraindications to giving thrombolytics. Answer B is also incorrect because a GI bleed is a contraindication only if it occurred within the last 3 months. Likewise, only a systolic BP greater than 180 mm Hg or a diastolic BP greater than 110 mm Hg is a contraindication for thrombolytics, making Answer D incorrect. Finally, this patient is not in cardiogenic shock (Answer E) because he does not have hypotension or other symptoms to suggest shock.

A 60-year-old woman with hypertension and diabetes mellitus presents with several weeks of exertional chest heaviness. Her medications include aspirin, a -blocker, an angiotensin-converting enzyme (ACE) inhibitor, and a statin. A treadmill test is ordered, and she develops chest pain and 3-mm ST-segment depressions at 4 minutes of exercise. She is referred for catheterization, which reveals a 90% proximal left anterior descending (LAD) stenosis, a 70% midcircumflex stenosis, and a 90% proximal right coronary artery (RCA) stenosis, with an ejection fraction of 40%. What should be recommended? A. Continued medical therapy B. Percutaneous transluminal coronary angioplasty (PTCA)/stenting of LAD, circumflex, and RCA C. Coronary artery bypass surgery D.

Addition of an oral glycoprotein IIb/IIIa (GP IIb/IIIa) inhibitor

Correct Answer: Coronary artery bypass surgery This question illustrates high-risk coronary anatomy and which patients derive mortality benefit from bypass surgery. These include patients with left main disease, patients with triple-vessel disease and left ventricular (LV) dysfunction (as in this example), patients with two- or three-vessel disease with proximal LAD involvement, and patients with three-vessel disease and ischemia at low workload (as in this example). This woman has high-risk anatomy and should be revascularized; therefore, Answer A is incorrect. Oral GP IIb/IIIa inhibitors are not approved for the treatment of coronary artery disease, and therefore, Answer D is also incorrect. For triple-vessel disease and LV dysfunction, surgery is usually a better choice than multivessel angioplasty (Answer B) because it offers greater survival benefit and a decreased need for recurrent revascularization compared with angioplasty. Diabetics also have a greater 5-year survival rate with surgery than with angioplasty.

A 70-year-old man with an acute inferior myocardial infarction (MI) develops cardiogenic shock. On examination, his jugular venous pressure (JVP) is elevated and his lungs are clear. A SwanGanz catheter is placed, which reveals a low cardiac index and pulmonary capillary wedge pressure (PCWP) of 6 mm Hg. Which of the following is the best treatment? A. IV nitroglycerin B. Surgery C. IV -blocker D. IV saline E. Dopamine

Correct Answer: IV saline This question illustrates an example of a right ventricular (RV) infarct. RV involvement typically occurs with inferior MI, and the classic triad on examination is hypotension, elevated JVP, and clear lungs. The problem is that the RV is unable to move fluid to

the left side of the heart, resulting in high right-sided pressures (increased JVP), low left-sided pressures (low PCWP and clear lungs), and systemic hypotension. These patients need fluid. Drugs that decrease preload even further (e.g., nitrates) are contraindicated (Answer A). At a time when the patient is in shock and the RV is contracting poorly, IV -blockers would not be a good choice (Answer C). Surgery is not necessary or helpful at this point (Answer B). Dopamine or another pressor is not required unless the hypotension is refractory to aggressive fluid resuscitation (Answer E). IV saline is the best treatment.

A 73-year-old man with a history of hypertension and hypercholesterolemia has experienced intermittent chest pain for years. He describes the pain as tightness in the center of his chest that predictably occurs after climbing three flights of stairs and is easily relieved with 1 sublingual nitroglycerin tablet. He notes no change in the pattern of his chest pain in the last 2 years. A prior exercise stress test was positive, but he refused further intervention. Regarding his medical condition, which medications have been shown to reduce mortality and risk of myocardial infarction? A. Angiotensin-converting enzyme (ACE) inhibitors only B. ACE inhibitors and aspirin C. ACE inhibitors, aspirin, and -blockers D. ACE inhibitors, aspirin, and nitrates E. ACE inhibitors, aspirin, and statins

Correct Answer: ACE inhibitors, aspirin, and statins Aspirin and statins reduce mortality and incidence of myocardial infarction (MI). The Heart Outcomes Prevention Evaluation trial also showed that ACE inhibitors reduce risk of MI, stroke, and death in patients with coronary artery disease. Nitrates relieve anginal symptoms but do not reduce risk of MI or improve survival. While -blockers are excellent for relieving symptoms, they have not been shown to reduce risk of MI or death in patients with chronic stable angina (they have been shown, however, to reduce mortality post-MI). Long-acting calcium channel blockers are second-line agents for the treatment of stable angina and may improve symptoms, but they do not improve survival. Short-acting dihydropyridines (e.g., nifedipine) may actually increase the risk of death and MI in patients with ischemic heart disease and are contraindicated in acute ischemic syndromes.

A 50-year-old woman presents to the emergency room with 25 minutes of substernal chest pressure. She has a history of hypertension. Her electrocardiogram reveals T-wave inversions in leads V1 through V6. Which of the following strategies is not appropriate? A. Give acetylsalicylic acid (ASA), heparin, -blocker, and nitrates B. Give ASA, heparin, -blocker, and nitrates, and add glycoprotein IIb/IIIa (GP IIb/IIIa) inhibitor if symptoms do not resolve rapidly C. Tissue plasminogen activator D. Notify the catheterization team even if her symptoms resolve

Correct Answer: Tissue plasminogen activator This question deals with the proper management of patients with unstable angina/nonST-segment elevation myocardial infarction (UA/NSTEMI). In the absence of ST-segment elevations or a new left bundle branch block, the woman in this example meets no criteria for thrombolytic therapy, and therefore Answer C is the inappropriate choice. Aspirin, heparin, -blockers, and nitrates are all indicated for the treatment of UA/NSTEMI, as are GP IIb/IIIa inhibitors. T-wave inversions across the precordium usually suggest disease in the left main or proximal left anterior descending artery, and it is reasonable to refer high-risk patients with UA/NSTEMI to the catheterization lab, even if their symptoms stabilize.

A 55-year-old man is admitted to the coronary care unit with an acute inferior myocardial infarction (MI). He reperfuses with tissue plasminogen activator and does well. Later that day he has an 8-beat run of unsustained ventricular tachycardia and begins showing roughly 6 premature ventricular contractions per minute on the monitor. How should you treat him? A. -Blocker B. Lidocaine C. Procainamide D. Amiodarone E. Calcium channel blocker

Correct Answer: -Blocker In the peri-infarct period, asymptomatic unsustained ventricular tachycardia (VT) and premature ventricular contractions should not be treated with antiarrhythmic therapy, as this has been associated with increased mortality. Unsustained VT in the first 24 hours after an infarct tends to be transient and benign. A -blocker is a good choice given the recent infarct and may also help control the ventricular ectopy.

A 65-year-old man with hypertension presents with the acute onset of knife-like chest pain radiating to his back. On examination, he appears anxious. His heart rate is 70 beats/min and his blood pressure is 150/100 mm Hg. His lungs are clear, and he has an audible diastolic murmur heard in the right upper sternal border. His electrocardiogram reveals 1-mm ST-segment elevations in inferior leads. A chest x-ray reveals clear lungs and a widened mediastinum. What should be done next? A. Treat with aspirin, heparin, and a -blocker B. Emergent percutaneous transluminal coronary angioplasty C. Give tissue plasminogen activator D. Start a -blocker and order a transesophageal echocardiogram E. Wait to see if the troponin is positive

Correct Answer: Start a -blocker and order a transesophageal echocardiogram This question illustrates that not all patients presenting with ST-segment elevations should be treated as thought they have a typical ST-segment elevation myocardial infarction. The person in this example has an acute aortic dissection (clues include the pain radiating to the back, the new murmur of aortic regurgitation, and the widened mediastinum). He has ST-segment elevations because the aorta has dissected back to one of the coronary arteries and not because of an acute coronary thrombosis that requires angioplasty (Answer B). Heparin (Answer A) and thrombolytics (Answer C) should be avoided. The best answer is D because -blockers lower blood pressure to help prevent further dissection, and transesophageal

echocardiogram is an excellent study for diagnosing an aortic dissection. This patient's best chance for survival is emergent surgery.

A 70-year-old man with hypertension comes to see you and describes 6 months of stable exertional angina. Despite the symptoms, he has remained active. His resting electrocardiogram (ECG) is normal. You order an exercise treadmill test (ETT). He exercises 8 minutes on a Bruce protocol and reaches 100% of his maximal predicted heart rate. He develops his usual anginal symptoms at peak exercise, and the ECG shows only nonspecific changes. What do you do next? A. Stress thallium test B. Dobutamine echocardiogram C. Refer for cardiac catheterization D. Medical therapy E. No intervention required

Correct Answer: Medical therapy This patient has chronic stable angina. The fact that he has had stable symptoms, has remained active, and has a baseline normal ECG are all good prognostic signs. In a 70-year-old male who has coronary risk factors and a typical history of exertional angina, the pretest probability of coronary artery disease (CAD) is high. The reason for ordering the stress test is not to determine whether or not he has CAD, but rather to assess his risk of MI and death (a negative test in a patient like this does not rule out CAD but does suggest that his risk for future events is low). The patient reaches 100% of his functional capacity and has anginal symptoms but has no ischemic ECG changes. For a low-risk patient like this one, who is on no medical therapy, the most appropriate next step is to begin medical therapy to reduce his symptoms (e.g., nitrates, -blockers) and to reduce mortality and incidence of MI (aspirin, lipidlowering drugs). Further stress testing (Answers A and B) will be unlikely to change case management, and stable patients determined to be at low risk should not be sent for catheterization (Answer C) until they have failed medical therapy.

You elect to treat the patient in the previous question medically. He returns a year later and reports that he is now getting chest pain more often and with less activity. You have been treating him with aspirin, a -blocker, and a statin for his cholesterol. On examination, heart rate (HR) is 55

beats/min and blood pressure is 120/80 mm Hg. Lab results show low-density lipoprotein cholesterol of 95 mg/dL. You repeat his stress test, and this time he develops chest pain and 2- to 3-mm ST-segment depressions in leads II, III, aVF, and V4 through V6 at 4 minutes into exercise. What should you do next? A. Prescribe a long-acting calcium channel blocker B. Increase his -blocker C. Refer for cardiac catheterization D. Admit to the hospital, start heparin, and consult cardiology for catheterization E. Leave things alone because he has been stable for a long time

Correct Answer: Refer for cardiac catheterization This patient with chronic angina is now failing medical therapy. Despite good control of his heart rate, blood pressure, and cholesterol, he is now experiencing chest pain more easily and with less activity. His exercise treadmill test (ETT) is worrying in that it is positive early (<6 minutes) and there are deep ST-segment depressions without much exercise. These findings suggest possible high-risk coronary anatomy (e.g., triple-vessel disease), which puts him at higher risk for future cardiac events and death. The reason to refer for catheterization is that you suspect high-risk anatomy that should be revascularized to relieve symptoms and improve survival. Because he is stable, his catheterization can be scheduled electively, and he does not need to be admitted urgently (Answer D). There is also no need for intravenous heparin (Answer D) in someone with stable exertional angina. Increasing his -blocker (Answer B) is probably unwise because his HR is already 55 beats/min. Long-acting calcium channel blockers (Answer A) are second-line agents for chronic stable angina and could be added, but catheterization is still warranted given his high-risk ETT and escalating symptoms.

Which of the following statements about an unstable plaque is false? A. An unstable plaque typically has a fibrous cap that covers a lipid-rich layer B. The plaques most likely to rupture are the ones that cause the most severe stenoses seen at the time of catheterization C.

A lipid-rich plaque causing a 50% stenosis is more likely to rupture than a calcified and fibrotic plaque causing a 90% stenosis D. Most deaths from myocardial infarction are associated with plaque rupture

Correct Answer: The plaques most likely to rupture are the ones that cause the most severe stenoses seen at the time of catheterization The extent of luminal narrowing at catheterization is often not predictive of which plaque will subsequently rupture. Usually, it is the smaller, lipid-rich, nonflow-limiting plaques that are most likely to rupture (Answer C). The larger fibrotic plaques tend to be more stable. Answers A and D are both true statements.

Which of the following statements is false with respect to ST-segment elevation myocardial infarction (STEMI) versus nonST-segment elevation myocardial infarction (NSTEMI)? A. There is a greater chance of finding an occlusive thrombus in a STEMI B. Patients with NSTEMI have a more developed coronary collateral network C. A person with a STEMI is more likely to have single-vessel disease than a person with an NSTEMI D. In the absence of percutaneous coronary intervention (PCI), glycoprotein IIb/IIIa (GP IIb/IIIa) inhibitors are not indicated for STEMI E. The thrombus in a STEMI is more platelet rich than is the thrombus of an NSTEMI

Correct Answer: The thrombus in a STEMI is more platelet rich than is the thrombus of an NSTEMI All of the statements are true, except Answer E. The thrombus of a STEMI tends to be richer in fibrin, whereas the thrombus of an NSTEMI is richer in platelets. This may be one important reason why thrombolytics are efficacious for STEMI, while GP IIb/IIIa inhibitors work better for NSTEMI. One should also remember that GP IIb/IIIa inhibitors are indicated for unstable angina and NSTEMI. In STEMI, unless PCI is performed, GP IIb/IIIa inhibitors are not indicated.

A 62-year-old man presents to the emergency department with intermittent substernal chest tightness for the last 2 days. The pain radiates to the left jaw, occurs with exertion, and is relieved by rest. The longest episode was 2 minutes. He is admitted to the hospital and rules out for a myocardial infarction. A subsequent stress test is positive. He undergoes cardiac catheterization, which reveals a 90% lesion in the right coronary artery and otherwise insignificant disease. All of the following antiplatelet agents are reasonable to use at the time of his percutaneous angioplasty except A. Aspirin B. Abciximab C. Eptifibatide D. Ticlopidine E. Clopidogrel

Correct Answer: Ticlopidine Unless a patient has an aspirin allergy, aspirin is always used with a percutaneous coronary intervention. Abciximab and eptifibatide are two glycoprotein IIb/IIIa (GP IIb/IIIa) inhibitors that have been shown to reduce ischemic events and improve survival when given during angioplasty procedures and are now frequently used in the catheterization lab. Ticlopidine and clopidogrel are antiplatelet agents that inhibit ADP-mediated platelet activation. These agents help decrease stent thrombosis. Because of associated neutropenia, ticlopidine is now used infrequently and has been essentially replaced by clopidogrel.

A 60-year-old man presents with an acute anterior myocardial infarction (MI). He receives tissue plasminogen activator in the emergency room 1 hour after onset of symptoms, with rapid resolution of pain and ST-segment elevations. He is admitted to the coronary care unit feeling well, pain free, and with stable blood pressure and pulse. Which intervention is least appropriate next? A. Urgent cardiac catheterization with possible percutaneous coronary intervention (PCI) B. Angiotensin-converting enzyme (ACE) inhibitor C. Heparin

D. -Blocker E. Daily aspirin

Correct Answer: Urgent cardiac catheterization with possible percutaneous coronary intervention (PCI)

This question emphasizes the point that there are no data to support that stable patients who rapidly reperfuse with thrombolytic therapy and have no manifestations of ischemia derive any mortality benefit from cardiac catheterization for PCI. Aspirin, heparin, ACE inhibitors, and -blockers are all indicated in an acute ST-segment elevation MI.

A 70-year-old man is admitted to the coronary care unit with an acute inferoposterior myocardial infarction (MI). He is treated with tissue plasminogen activator and initially does well. On day 3, however, he suddenly becomes hypotensive, and you are called to see him. On examination, he is lying flat in bed with a pulse of 80 beats/min, respiratory rate of 14 breaths/min, and systolic blood pressure of 80 mm Hg. He has an elevated jugular venous pressure (JVP), clear lungs, a new pansystolic murmur, and a palpable thrill. Which of the following is correct? A. The triad of clear lungs, elevated JVP, and hypotension following a posterior MI suggests a right ventricular infarction, and he should receive fluids B. Sudden hypotension with elevated JVP in a patient who has been lying in bed for several days suggests a pulmonary embolism; the new systolic murmur is likely tricuspid regurgitation in the setting of sudden increase in pulmonary pressure C. This presentation is consistent with rupture of the ventricular septum and cardiac surgery is necessary D. This presentation is consistent with rupture of a papillary muscle, and cardiac surgery is necessary E. This presentation is consistent with post-MI pericarditis (Dressler syndrome) and prednisone should be administered

Correct Answer: This presentation is consistent with rupture of the ventricular septum and cardiac

surgery is necessary A new systolic murmur following an acute MI could represent several things, and the differential diagnosis is outlined by the various answers given as choices. Hypotension, right ventricular (RV) failure, a new systolic murmur, and a palpable thrill are all most consistent with rupture of the ventricular septum. This typically occurs several days following the infarct. Answer A is incorrect because RV infarcts do not cause sudden hypotension several days after initial presentation. They also do not cause a palpable thrill. Answer B is incorrect because a pulmonary embolism that causes hypotension would also be expected to cause tachycardia and tachypnea and would not cause a thrill. Answer D is incorrect because acute papillary muscle rupture causing severe mitral regurgitation would result in pulmonary edema, and the patient would not have clear lungs or be able to lie flat. Answer E is also incorrect because Dressler syndrome is usually associated with fever and a rub, not a thrill and hypotension.

Match the problem with its hemodynamics:* *CI, cardiac index; PaO2 sat, pulmonary artery oxygen saturation; PCWP, pulmonary capillary wedge pressure; RA, right atrial pressure; and RV, right ventricular pressure. A. Papillary muscle rupture

Please pick the matching Item....


B. Right ventricular infarct

Please pick the matching Item....


C. Rupture of the ventricular septum

Please pick the matching Item....


D. Tamponade A. RA = 21 mm Hg, RV = 60/22 mm Hg, PCWP = 20 mm Hg, CI = 1.7 L/min/m2, PaO2 sat = 70% B. RA = 15 mm Hg, RV = 60/30 mm Hg, PCWP = 6 mm Hg, CI = 1.7 L/min/m2, PaO2 sat = 70% C. RA = 12 mm Hg, RV = 60/30 mm Hg, PCWP = 10 mm Hg, CI = 1.7 L/min/m2, PaO2 sat = 95% D. RA = 12 mm Hg, RV = 50/20 mm Hg, PCWP = 40 mm Hg, CI = 1.6 L/min/m2, PaO2 sat = 75%

Correct Answer: RA = 12 mm Hg, RV = 50/20 mm Hg, PCWP = 40 mm Hg, CI = 1.6 L/min/m2, PaO2 sat = 75%, RA = 12 mm Hg, RV = 60/30 mm Hg, PCWP = 10 mm Hg, CI = 1.7 L/min/m2, PaO2 sat = 95%, RA = 15 mm Hg, RV = 60/30 mm Hg, PCWP = 6 mm Hg, CI = 1.7 L/min/m2, PaO2 sat = 70%, RA = 21 mm Hg, RV = 60/22 mm Hg, PCWP = 20 mm Hg, CI = 1.7 L/min/m2, PaO2 sat = 70% 1:D, 2:C, 3:B, 4:A. Papillary muscle rupture is Answer D. With the acute mitral regurgitation, there is a huge increase in left atrial pressure (which is reflected in the high PCWP). Some of the oxygenated blood from the left ventricle may even jet back into the pulmonary artery, giving the slightly higher O2 saturation. Right ventricular infarct is Answer B because, with a right ventricular infarct, the right-sided pressures are high while left-sided (wedge) pressures remain low. Rupture of the septum is Answer C, which reveals high right-sided pressures, fairly normal wedge pressure, and increased pulmonary artery O2 saturation (oxygen-rich blood moves from the left side of the heart to the right). Tamponade is Answer A because there is equalization of right- and left-sided diastolic pressures.

A 22-year-old man with no significant past history presents to the emergency room complaining of intense, left-sided chest pain radiating to his left arm. He admits to smoking crack cocaine about 2 hours ago. His pain has now been present for over an hour. His physical examination reveals a pale, diaphoretic man. His blood pressure is 155/90 mm Hg. His pulse is 110 beats/min. Pulmonary and cardiac exams are only notable for tachycardia. His electrocardiogram reveals 2mm ST-segment elevation in the inferior leads. The troponin level is elevated. All of the following are appropriate treatment measures for this patient except A. Oxygen B. Lorazepam C. Propranolol D. Nitroglycerin E. Aspirin

Correct Answer: Propranolol This patient presents with evidence of a cocaine-induced myocardial infarction. Chest pain can occur minutes to hours after cocaine use. It is thought to cause myocardial

ischemia by increasing myocardial oxygen demand, causing vasoconstriction and/or accelerating thrombosis. All of the answers listed except for propranolol are appropriate therapies for cocaine-induced infarction. Since cocaine-induced vasoconstriction is primarily caused by stimulation of -adrenergic receptors, blockade may actually potentiate the vasoconstrictor response by leaving unopposed -mediated constriction. Benzodiazepines can serve to reduce the heart rate and systemic arterial pressure. In animal studies, they have been shown to attenuate cocaine's effect on the heart.

A 56-year-old man who is worried about "high cholesterol" comes to your office for cardiac risk assessment. He is a lifelong nonsmoker and has a blood pressure of 125/82 mm Hg on amlodipine 5 mg and an unremarkable physical exam. His total cholesterol is 218 mg/dL and high-density lipoprotein cholesterol is 47 mg/dL. His electrocardiogram is unremarkable. He is on no cholesterol medication but does not eat red meat or fatty foods. He is not overweight, and he walks 2 miles on his treadmill 6 days a week. He has recently decided to take a baby aspirin to prevent heart attacks. He is interested in knowing whether he should do more to reduce his risk for heart disease. You calculate his Framingham risk score and determine that he has a 12% risk of coronary heart disease events at 10 years. What do you do next? A. Reassure him B. Recommend that he increase his exercise regimen by adding weight training C. Schedule him for an exercise stress test D. Start him on atorvastatin 40 mg a day E. Schedule him for a calcium score by computed tomography

Correct Answer: Schedule him for a calcium score by computed tomography This patient has an intermediate Framingham risk score and would benefit from further risk stratification to determine if more aggressive therapy is warranted. He is already exercising regularly and there is no benefit to adding weight training. Exercise stress testing could be performed but will likely not be helpful in further risk stratification in an asymptomatic individual who is physically active. Starting him on atorvastatin without further stratification is a viable option, but the dose of 40 mg is not the appropriate starting dose for a total cholesterol of 218 mg/dL. Assessment of coronary calcium will give an estimate of this burden of coronary plaque, and a very low or 0 calcium score can reliably reassure the patient that he is at very low risk for

future coronary events. Presence of a high score would increase his risk for coronary events and would indicate that he would benefit from addition of a statin.

A 50-year-old man with a 10-year history of type 2 diabetes, hypertension, obesity, prior myocardial infarction (MI), and transient ischemic attack (TIA) is admitted from the emergency room (ER) with chest pain and a troponin I of 3. He is currently pain-free on aspirin, oral metoprolol, and intravenous nitroglycerin and heparin. His vitals are pulse 95 of beats/min, blood pressure of 188/95 mm Hg, and pulse oximetry of 97% on room air. An electrocardiogram shows new 1-mm ST-segment depressions, and an echocardiogram performed in the ER shows an ejection fraction of 40% and a new anterior wall motion abnormality. His Thrombolysis In Myocardial Infarction (TIMI) score puts him in a high-risk category, so an early invasive strategy is indicated. As part of this, in anticipation of impending coronary angiography and as you are planning to escort him to the catheterization laboratory, what is the next best step? A. Start eptifibatide B. Start clopidogrel loading C. Start prasugrel loading D. A and B E. A and C

Correct Answer: A and B This is a young patient with a high TIMI risk score, prior MI, and now a new nonSTsegment elevation myocardial infarction. As part of initiating triple-antiplatelet therapy in high-risk subjects going for early invasive strategy, he is a candidate for glycoprotein IIb/IIIa inhibitor and a thienopyridine in anticipation of percutaneous coronary intervention. The choice of prasugrel is worth considering as he does have high-risk features, particularly a prior MI and diabetes; however, the history of prior TIA is a contraindication (along with active bleeding) due to excessive risk of major bleeding. Furthermore, this patient may be a candidate for coronary artery bypass grafting, which would further increase risk of major bleeding if prasugrel were started.

Вам также может понравиться